Mathcenter Forum

Mathcenter Forum (https://www.mathcenter.net/forum/index.php)
-   ข้อสอบในโรงเรียน ม.ปลาย (https://www.mathcenter.net/forum/forumdisplay.php?f=21)
-   -   ข้อสอบ entrance (https://www.mathcenter.net/forum/showthread.php?t=1589)

ฝันบรรเจิด 01 พฤศจิกายน 2005 13:25

ข้อสอบ entrance
 
ใครมีวิธีคิดข้อสอบพวกนี้แบบไม่แปลกๆ หรือ แปลกๆ ช่วยฉลยด้วยนะครับ

มี.ค.47

16.กำหนดให้ z1,z2,z3 เป็นจำนวนเชิงซ้อน ซึ่งสอดคล้องกับเงื่อนไข

(z1)(z2)(z3)=1 และ z1+z2+z3= (1/z1)+(1/z2)+(1/z3)

พิจารณาข้อความต่อไปนี้

1) (1-z1)(1-z2) = { 1-(1/z1)}{1-(1/z2)}

2) ถ้า z1 ไม่เท่ากับ 1 และ z2 ไม่เท่ากับ 1 แล้ว | z3 + i|| z3 - i | = 4

ข้อใดถูก
.............................................
ต.ค.45

17) กำหนดให้ z1,z2,z3 เป็นจำนนเชิงซ้อนซึ่งมีคุณสมบัติว่า |z1|= |z2|=|z3|= 1

,z1+z2+z3= 0 และให้ Re(Z) แทนส่วนจริงของจำนวนเชิงซ้อน z พิจารณาข้อความต่อไปนี้

1) ส่วนจริงของผลคูณของ z1 และ สังยุคของ z2 = 1/2

2) | z1 - z2 |= 3^(1/2)

ข้อใดถูก
........................................
มี.ค.45

17) กำหนดให้ ABC เป็นสามเหลี่ยมที่มีคุณสมบัติว่า 5|เวกเตอร์ AB| = |เวกเตอร์ BC|+|เวกเตอร์ CA| ถ้า M และ N เป็นจุดแบ่งครึ่งด้าน BC และ AC ตามลำดับ แล้ว พิจารณาข้อความต่อไปนี้

1) เวกเตอร์ MN = (1/2)( เวกเตอร์BC-เวกเตอร์AC)

2) (เวกเตอร์AM) dot (เวกเตอร์BN) =0

ข้อใดถูก
......................................

ใครเฉลยๆ ได้หลายๆ วิธีช่วยกันคิดนะครับ ขอบคุง..^^ :p :p :p :p :p :p

M@gpie 01 พฤศจิกายน 2005 15:38

3 ข้อนี้ เรียกได้ว่าข้อปราบเซียนเลยก็ว่าได้ครับ เพราะถ้าจะคิดให้เป็นหลักเป็นการจริงๆแล้วยากมากที่จะทำทันในห้องสอบ (ตอนผมสอบก็ทำไม่ได้ :( )
เอาที่คิดออกก่อนนะคับ

ข้อ 17 ) ตค. 45
ข้อนี้ส่วนใหญ่จะเฉลยโดยว่า \( z_1 , z_2 ,z_3 \) เป็นรากที่ 3 ของหนึ่ง แล้วก็ลองแทน ค่าไปดู ถ้าไม่ทำแบบนี้ก็สามารถทำได้ดังนี้ครับ
เนื่องจาก \( z_1 + z_2 +z_3 = 0 \) จะได้ว่า \( z_1^{*} + z_2^{*} +z_3^{*} = 0 \) เช่นกัน
ดังนั้น เปลี่ยนสมการใหม่เป็น \( z_1 + z_2 - z_3 = 0 \) และ \( z_1^{*} + z_2^{*} = - z_3^{*} \)
จับสองสมการมาคูณกันจะได้ \( (z_1 + z_2)(z_1^{*} +z_2^{*}) = z_3 z_3^{*} \)
\( \mid z_1 \mid ^2 + (z_1 z_2^{*} + z_1^{*}z_2) +\mid z_2 \mid ^2 = \mid z_3 \mid ^2 \)
จะได้ว่า \( z_1 z_2^{*} + z_1^{*}z_2 = 2Re(z_1 z_2^{*}) = -1 \)
ดังนั้น \( Re(z_1 z_2^{*}) = - \frac{1}{2} \)
ตอบ ข้อ 1 ผิด

ต่อไปใช้ผลจากข้อ 1
พิจารณา \( \mid z_1 - z_2 \mid ^2 = (z_1 - z_2)(z_1^{*} - z_2^{*}) = \mid z_1 \mid ^2 - (z_1 z_2^{*} + z_1^{*}z_2) +\mid z_2 \mid ^2 = 3 \)
ดังนั้น \(\mid z_1 -z_2 \mid = \sqrt{3} \)
ข้อ 2 ถูกต้อง

deathspirit 01 พฤศจิกายน 2005 21:42

จาก\( z_{1}z_{2}z_{3}=1 \)
จะได้\( (1-\frac{1}{z_{1}})(1-\frac{1}{z_{2}})=(z_{1}z_{2}z_{3}-z_{2}z_{3})(z_{1}z_{2}z_{3}-z_{1}z_{3}) \)
\( =z_{2}z_{3}(z_{1}-1)z_{1}z_{3}(z_{2}-1) \)
\( =z_{3}(1-z_{1})(1-z_{2}) \)

tunococ 02 พฤศจิกายน 2005 03:35

มี.ค.45 น่าจะผิดทั้งสองข้อนา ...

passer-by 02 พฤศจิกายน 2005 19:55

ข้อที่เป็น เวกเตอร์ น่าจะถูกเฉพาะข้อ 1 ข้อเดียวนะครับ

ข้อ 1 ความหมายเทียบเท่ากับเวกเตอร์ MN คือครึ่งหนึ่งของเวกเตอร์ BA ซึ่งเป็นจริง ตาม ทฤษฎีบท ทาง plane geometry อยู่แล้ว ที่บอกว่า เส้นที่ลากเชื่อม midpoint ของ ด้าน 2 ด้านของสามเหลี่ยม จะขนานกับด้านที่ 3 และยาวเป็นครึ่งหนึ่งของด้านที่ 3

ส่วนข้อ 2 จะถูก ก็ต่อเมื่อ \(\large 5|\vec{AB}|^{2}= |\vec{BC}|^{2}+|\vec{CA}|^{2} \) ซึ่งทำได้โดย เขียนเวกเตอร์ AM ,BN ในเทอมของด้านบางด้านของสามเหลี่ยม แล้ว dot กัน

deathspirit 07 พฤศจิกายน 2005 21:17

มีนา47 ข้อ16.1

ถ้าเกิดว่าทำแบบนี้ จะได้ว่าถูกครับ (แต่อีกอันด้านบนที่ทำไว้ ได้ว่าผิด หรือแสดงว่า \( z_{3}=1 \) ก็ไม่รู้)

จาก \(z_{1}+z_{2}+z_{3}=\frac{1}{z_{1}}+\frac{1}{z_{2}}+\frac{1}{z_{3}}\)
\(= \frac{z_{2}z_{3}+z_{1}z_{3}+z_{1}z_{2}}{z_{1}z_{2}z_{3}}\)
\(= z_{2}z_{3}+z_{1}z_{3}+z_{1}z_{2} \)

จาก \((1-z_{1})(1-z_{2}) = (1-\frac{1}{z_{1}})(1-\frac{1}{z_{2}})\)
\(1-z_{2}-z_{1}+z_{1}z_{2} = 1-\frac{1}{z_{1}}-\frac{1}{z_{2}}+\frac{1}{z_{1}z_{2}}\)
\(z_{3}-z_{2}z_{3}-z_{1}z_{3}-z_{1}z_{2}+z_{1}z_{2} = -\frac{1}{z_{1}}-\frac{1}{z_{2}}+\frac{1}{z_{1}z_{2}} \)
\(\frac{1}{z_{1}z_{2}}-\frac{z_{2}}{z_{1}z_{2}}-\frac{z_{1}}{z_{1}z_{2}} = -\frac{z_{2}}{z_{1}z_{2}}-\frac{z_{1}}{z_{1}z_{2}}+\frac{1}{z_{1}z_{2}}\) ถูกต้อง

M@gpie 07 พฤศจิกายน 2005 23:06

ได้วิธีคิดละครับ ลองตรวจสอบกันดู ยากจิงๆด้วยนะเนี่ย เหอๆ ใครมีวิธีที่ดีกว่าเสนอได้เลยนะครับ
จากโจทย์จะได้เงื่อนไขมาสองอันคือ
\(z_1z_2z_3 = 1 \) และ \( z_1 +
z_2 +z_3 = \frac{1}{z_1} + \frac{1}{z_2} + \frac{1}{z_3} = z_1z_2+z_2z_3+z_1z_3 = a \)
เราจะได้พหุนามที่มี \( z_1,z_2,z_3\) เป็นคำตอบของสมการคือ
\( x^3 - ax^2 +ax -1 = 0 \)
แยกตัวประกอบพหุนามจะได้ว่า \( (x-1)(x^2+(1-a)x+1)=0 \)
จากสมการสรุปได้ว่า \( z_1 , z_2 , z_3 \) ต้องมีตัวใดตัวหนึ่งมีค่าเป็น 1 แน่นอน

กรณี \( z_3 \neq 1 \) จะได้ \( z_1=1 \) หรือ \( z_2=1 \) จึงได้ว่า ข้อ 1). ถูก

กรณี \( z_3 = 1 \) จะได้ ความสัมพันธ์ที่เหลือคือ \( z_1z_2 =1 \) และ \( z_1+z_2= \frac{1}{z_1} + \frac{1}{z_2} \)
ซึ่งทำให้ ข้อ 1) ถูกเช่นกัน

ข้อ 2) เนื่องจากโจทย์ให้ \( z_1 \neq 1 , z_2 \neq 2 \) จะได้ว่า \( z_3 = 1 \) แทนค่าในโจทย์พบว่าผิด

warut 22 มกราคม 2006 18:58

อ้างอิง:

ข้อความเดิมของคุณ M@gpie:
ข้อ 17 ) ตค. 45
ข้อนี้ส่วนใหญ่จะเฉลยโดยว่า \( z_1 , z_2 ,z_3 \) เป็นรากที่ 3 ของหนึ่ง

เฉลยแบบนั้นไม่ถูกครับ ยกตัวอย่างเช่น ถ้าให้\[z_1=-1, z_2=e^{\pi i/3}, z_3=e^{-\pi i/3}\]แม้ว่า \(z_1,z_2,z_3\) จะไม่ใช่รากที่สามของ 1 แต่เราก็ยังคงได้ว่า \(|z_1|= |z_2|= |z_3|= 1\) และ \(z_1+z_2+z_3=0\) ครับ


เวลาที่แสดงทั้งหมด เป็นเวลาที่ประเทศไทย (GMT +7) ขณะนี้เป็นเวลา 09:03

Powered by vBulletin® Copyright ©2000 - 2024, Jelsoft Enterprises Ltd.
Modified by Jetsada Karnpracha